Line 62: Line 62:
 
[[Category: GATE2003]]
 
[[Category: GATE2003]]
 
[[Category: Automata Theory]]
 
[[Category: Automata Theory]]
[[Category: Previous year GATE questions]]
+
[[Category: Automata  questions]]

Revision as of 07:06, 15 April 2014

Define languages L0 and L1 as follows :

$L_0 = \{< M, w, 0 > |$ $M$ halts on $w\} $

$L_1 = \{< M, w, 1 > |$ $M$ does not halts on $w\}$

Here $< M, w, i >$ is a triplet, whose first component. $M$ is an encoding of a Turing Machine, second component,$ w$, is a string, and third component, $i$, is a bit.

Let $L = L_0 ∪ L_1$. Which of the following is true ?

(A) $L$ is recursively enumerable, but is not

(B) $L$ is recursively enumerable, but $ L'$ is not

(C) Both $L$ and $L'$ are recursive

(D) Neither $L$ nor $L'$ is recursively enumerable

Solution by Arjun Suresh

Both $L$ and $Lʼ$ are undecidable. Because halting problem can be solved with both $L$ and $Lʼ$.

Halting problem can be stated as follows: A machine $M$ and a word $w$ are given. You have to tell, if $M$ halts on $w$.

So, to solve halting problem $<M,w>$ using $L$, just give $<M,w,0>$ and $<M,w,1>$ to two instances of $T$ which is the Turing machine for $L$. If $T$ accepts the triplet $<M,w,0>$, it means $M$ halts on $w$ => we have solved halting problem. If $T$ accepts the triplet $<M,w,1>$, it means $M$ doesn't halt on $w$ => we have solved halting problem. We know that either $<M,w,0>$ or $<M,w,1>$ is in $L$. So, if $L$ is recursively enumerable, $T$ is bound to stop on at least one of these inputs (<math>TM</math> for a recursively enumerable language stops and accepts, when provided with a word in its language).

Hence, using $L$ we can solve halting problem => $L$ is not recursively enumerable. Similarly, we can also show that halting problem can be solved with $Lʼ$.

Hence, neither $L$ nor $Lʼ$ is recursively enumerable.



blog comments powered by Disqus

Define languages L0 and L1 as follows :

$L_0 = \{< M, w, 0 > |$ $M$ halts on $w\} $

$L_1 = \{< M, w, 1 > |$ $M$ does not halts on $w\}$

Here $< M, w, i >$ is a triplet, whose first component. $M$ is an encoding of a Turing Machine, second component,$ w$, is a string, and third component, $i$, is a bit.

Let $L = L_0 ∪ L_1$. Which of the following is true ?

(A) $L$ is recursively enumerable, but is not

(B) $L$ is recursively enumerable, but $ L'$ is not

(C) Both $L$ and $L'$ are recursive

(D) Neither $L$ nor $L'$ is recursively enumerable

Solution by Arjun Suresh[edit]

Both $L$ and $Lʼ$ are undecidable. Because halting problem can be solved with both $L$ and $Lʼ$.

Halting problem can be stated as follows: A machine $M$ and a word $w$ are given. You have to tell, if $M$ halts on $w$.

So, to solve halting problem $<M,w>$ using $L$, just give $<M,w,0>$ and $<M,w,1>$ to two instances of $T$ which is the Turing machine for $L$. If $T$ accepts the triplet $<M,w,0>$, it means $M$ halts on $w$ => we have solved halting problem. If $T$ accepts the triplet $<M,w,1>$, it means $M$ doesn't halt on $w$ => we have solved halting problem. We know that either $<M,w,0>$ or $<M,w,1>$ is in $L$. So, if $L$ is recursively enumerable, $T$ is bound to stop on at least one of these inputs (<math>TM</math> for a recursively enumerable language stops and accepts, when provided with a word in its language).

Hence, using $L$ we can solve halting problem => $L$ is not recursively enumerable. Similarly, we can also show that halting problem can be solved with $Lʼ$.

Hence, neither $L$ nor $Lʼ$ is recursively enumerable.



blog comments powered by Disqus